You are on page 1of 30

fiziks

Institute for NET/JRF, GATE, IIT‐JAM, M.Sc. Entrance, JEST, TIFR and GRE in Physics 
 
Modern Physics
IIT-JAM 2005
Q1. If M e , M p and M H are the rest masses of electron, proton and hydrogen atom in the

ground state (with energy 13.6 eV ), respectively, which of the following is exactly true?
( c is the speed of light in free space)
(a) M H  M p  M e

13.6 eV
(b) M H  M p  M e 
c2
13.6 eV
(c) M H  M p  M e 
c2
13.6 eV
(d) M H  M p  M e  K , where K   or zero
c2
Ans. : (c)

Solution: B.E.   M p  M e  M H  c 2  M H  M p  M e 
B.E.
where B.E.  13.6eV .
c2
IIT-JAM 2006
Q2. Electrons of energy E coming from x   impinge upon a potential barrier of width
2a and height V0 centered at the origin with V0  E , as shown in the figure below. Let

2mV0  E 
k . In the region a  x  a , the electrons is a linear combination of

V

V0

a a x

(a) e kx and e  kx (b) e ikx and e  kx (c) e ikx and e ikx (d) e ikx and e kx
Ans. : (a)
Solution: Since, V0  E in region a  x  a . Thus, Schrodinger equation is given by

H.No. 40-D, Ground Floor, Jia Sarai, Near IIT, Hauz Khas, New Delhi-110016
Phone: 011-26865455/+91-9871145498
Website: www.physicsbyfiziks.com | Email: fiziks.physics@gmail.com

fiziks
Institute for NET/JRF, GATE, IIT‐JAM, M.Sc. Entrance, JEST, TIFR and GRE in Physics 
 
 
2 2
  2m(Vo  E )
2
 2
  V   E     0   k 2  0
2m x 2 x 2 x 2
o
2
2mV0  E 
where k  .

Thus, the solution of the wave equation is e kx and e  kx , which is exponential in nature.
Q3. The relation between angular frequency  and wave number k for given type of waves
is  2   k   k 3 . The wave number k 0 for which the phase velocity equals the group
velocity is,
 1   1 
(a) 3 (b)   (c) (d)  
 3   2 
Ans. : (c)
d 
Solution: Group velocity, Vg  and phase velocity is V p 
dk k
 2 =  k + k 3 ………(A)
d
Differentiating both sides we get 2.    3 k 2
dk
Now dividing both sides by k we will get
 d  
2 .   3 k  2V p .Vg   3 k
k dk k k
For k  k0 and V p  Vg
1
   3 k0  2
2V   3 k0  V p  
2
p  
k0  2k0 2 
1
   2
From equation (A) V p      k0 
k  k0 
1 1
  3 k 0  2
  2
  k0 
Thus,        k0     0  k0 
 2k0 2   k0  2 k0 2 

H.No. 40-D, Ground Floor, Jia Sarai, Near IIT, Hauz Khas, New Delhi-110016
Phone: 011-26865455/+91-9871145498
Website: www.physicsbyfiziks.com | Email: fiziks.physics@gmail.com

fiziks
Institute for NET/JRF, GATE, IIT‐JAM, M.Sc. Entrance, JEST, TIFR and GRE in Physics 
 
Q4. A particle of rest mass m0 is moving uniformly in a straight line with relativistic velocity
 c , where c is the velocity of light in vacuum and 0    1 . The phase velocity of the
de Broglie wave associated with the particle is,
c c
(a)  c (b) (c) c (d) 2
 
Ans. : (b)
Solution: E 2  p 2 c 2  m02 c 4

dE E c2 c2 c
2E  2 pc 2  E.vg  pc 2    vp  vp  
dp p vg c 
Q5. A neutron of mass, mn  1027 kg is moving inside a nucleus to be a cubical box of size

1014 m with impenetrable walls. Take   1034 Js and 1MeV  1013 J . An estimate of
the energy in MeV of the neutron is,
1 1
(a) 80 MeV (b) MeV (c) 8 MeV (d) MeV
8 80
Ans:

 
2
3 2  2 3 10  1034 3 10  1068
Solution: E   
2mn a 2 2 1027  1014
  2 1027  1028
2

 15  1013 J  15 1013  1013 MeV  15MeV


IIT-JAM 2007
Q6. The following histogram represents the binding energy per particle ( B.E./ A ) in MeV as
a function of the mass number A of a
nucleus. A nucleus with mass number
8
A  180 fissions into two nuclei of
6
equal masses. In this process B.E.
A 4
(a) 180 MeV of energy is released
(b) 180 MeV of energy is absorbed 2
(c) 360 MeV of energy is released
40 80 120 160 200
(d) 360 MeV of energy is absorbed
A

H.No. 40-D, Ground Floor, Jia Sarai, Near IIT, Hauz Khas, New Delhi-110016
Phone: 011-26865455/+91-9871145498
Website: www.physicsbyfiziks.com | Email: fiziks.physics@gmail.com

fiziks
Institute for NET/JRF, GATE, IIT‐JAM, M.Sc. Entrance, JEST, TIFR and GRE in Physics 
 
Ans. : (c)
A A
Solution: A   or 180  90  90
2 2
Product B.E  90  6  90  6  1080 MeV
B.E. of nucleus A  180  4  720 MeV
Since, B.E of the product nucleus is greater than the nucleus A, hence in this process
energy is released and that is  1080  720 MeV  360 MeV .
Q7. The black body spectrum of an object O1 is such that its radiant intensity (i.e., intensity

per unit wavelength interval) is maximum at a wavelength of 200 nm . Another object O2

has the maximum radiant intensity at 600 nm . The ratio of power emitted per unit area by
O1 to that of O2 is

1 1
(a) (b) (c) 9 (d) 81
81 9
Ans. : (d)
1 T2 T1
Solution: From Wein’s law T  k , where k is a constant. Thus,  3
2 T1 T2

P1 T14
Power ( P ) is proportional to T 4    81
P2 T24
Q8. A particle is confined in a one dimensional box with impenetrable walls at x   a . Its
energy eigenvalue is 2 eV and the corresponding eigenfunction is as shown below.

a 0 a
The lowest possible energy of the particle is
(a) 4 eV (b) 2 eV (c) 1eV (d) 0.5eV
Ans. : (d)
Solution: The given state is representation of first exited state whose energy is 2 eV .

If En is energy of nth state and E0 is energy of ground state then, En  n 2 E0 .

So, E2  4 E0  2eV  E0  0.5eV

H.No. 40-D, Ground Floor, Jia Sarai, Near IIT, Hauz Khas, New Delhi-110016
Phone: 011-26865455/+91-9871145498
Website: www.physicsbyfiziks.com | Email: fiziks.physics@gmail.com

fiziks
Institute for NET/JRF, GATE, IIT‐JAM, M.Sc. Entrance, JEST, TIFR and GRE in Physics 
 
IIT-JAM 2008
Q9. A photon of wavelength  is incident on a free electron at rest and is scattered in the
backward direction. The functional shift in its wavelength in terms of the Compton
wavelength  c of the electron is,
C 2 C 3C 2 C
(a) (b) (c) (d)
2 3 2 
Ans. : (d)
Solution:   c (1  cos  )

When photon scattered in backward direction then    . So,   2c

 2C
Functional shift is =
 
Q10. In an inertial frame S , a stationary rod makes an angle  with the x -axis. Another
inertial frame S  moves with a velocity v with respect to S along the common x - x
axis. As observed from S  the angle made by the rod with the x - axis is   . Which of
the following statement is correct?
(a)    
(b)    
(c)     if v is negative and     if v is positive
(d)     if v is negative and     if v is positive
Ans. : (b)

v2
Solution: lx  l0 cos  1  , l y  l0 sin 
c2
ly tan 
tan       
lx v2
1 2
c

H.No. 40-D, Ground Floor, Jia Sarai, Near IIT, Hauz Khas, New Delhi-110016
Phone: 011-26865455/+91-9871145498
Website: www.physicsbyfiziks.com | Email: fiziks.physics@gmail.com

fiziks
Institute for NET/JRF, GATE, IIT‐JAM, M.Sc. Entrance, JEST, TIFR and GRE in Physics 
 
Q11. The activity of a radioactive sample is decreased to 75% of the initial value after 30 days.
The half-life (in days) of the sample is approximately
[You may use ln 3  1.1, ln 4  1.4 ]
(a) 38 (b) 45 (c) 59 (d) 69
Ans. : (d)

1  R  1  R0  1 4 1
Solution:   ln 0   ln   ln   1.4  1.1  1
t  R  30  3 / 4 R0  30  3  30 100

0.693 0.693
T1/ 2    69.3 day.
 1/100
IIT-JAM 2009
Q12. A wave packet in a certain medium is constructed by superposing waves of frequency 
around 0  100 and the corresponding wave-number k with k0  10 as given in the
table below,
 k
81.00 9.0
90.25 9.5
100.00 10.0
110.25 10.5
121.00 11.0
Find the ratio vg / v p of the group velocity vg and the phase velocity v p .

1 3
(a) (b) 1 (c) (d) 2
2 2
Ans. : (d)
0
Solution: For   0  100 and k  k0  10 the phase velocity is v p  =10
k0

 2  1 110.25  90.25
The group velocity is vg   =  20
k k2  k1 10.5  9.5

vg 20
 2
vp 10

H.No. 40-D, Ground Floor, Jia Sarai, Near IIT, Hauz Khas, New Delhi-110016
Phone: 011-26865455/+91-9871145498
Website: www.physicsbyfiziks.com | Email: fiziks.physics@gmail.com

fiziks
Institute for NET/JRF, GATE, IIT‐JAM, M.Sc. Entrance, JEST, TIFR and GRE in Physics 
 
Q13. Two spherical nuclei have mass numbers 216 and 64 with their radii R1 and R2 ,

R1
respectively. The ratio is
R2
(a) 1.0 (b) 1.5 (c) 2.0 (d) 2.5
Ans. : (d)
1/ 3
A 
1/ 3
R  216  6
Solution: 1   1      1.5
R2  A2   64  4

IIT-JAM 2010
Q14. A particle of mass m is confined in a two-dimensional infinite square well potential of
25 2  2
side a . The eigen-energy of the particle in a given state is E  . The state is
ma 2
(a) 4 -fold degenerate (b) 3 -fold degenerate
(c) 2 -fold degenerate (d) Non-degenerate
Ans. : (d)
Solution: The eigen-energy of the particle in a given state is given by
 22
E (nx2  n y2 ) where nx  1, 2,3... n y  1, 2, 3...
2ma 2
25 2  2
E can be obtained by nx  5 and n y  5 which is non degenerate .
ma 2

Q15. For a wave in a medium the angular frequency  and the wave vector k are related by,
 2   02  c 2 k 2  , where  0 and c are constants. The product of group and phase
velocities, i.e., v g .v p is

(a) 0.25 c 2 (b) 0.4 c 2 (c) 0.5 c 2 (d) c 2


Ans. : (d)
Solution:  2   02  c 2 k 2 
d  d
2  2c 2 k    c 2  v p .vg  c 2
dk k dk

H.No. 40-D, Ground Floor, Jia Sarai, Near IIT, Hauz Khas, New Delhi-110016
Phone: 011-26865455/+91-9871145498
Website: www.physicsbyfiziks.com | Email: fiziks.physics@gmail.com

fiziks
Institute for NET/JRF, GATE, IIT‐JAM, M.Sc. Entrance, JEST, TIFR and GRE in Physics 
 
1
Q16. Three identical non-interacting particles, each of spin and mass m , are moving in a
2
one-dimensional infinite potential well given by,
0 for 0  x  a
V  x  
 for x  0 and x  a
The energy of the lowest energy state of the system is
 2 2 2 2  2 3 2  2 5 2  2
(a) (b) (c) (d)
ma 2 ma 2 ma 2 2ma 2
Ans. : (c)
1
Solution: Spin s  means particles are fermions and it will obey Pauli Exclusion Principle.
2
Degeneracy, g  2s  1  g  2 means in every state maximum 2 identical particle can
be adjusted. If we have three fermions, then in ground state two fermions will be adjusted
and one fermion in next higher level will be adjusted. Thus, the energy of the lowest
 22 4 2  2 6 2  2 3 2  2
energy state of the system is 2    =
2ma 2 2ma 2 2ma 2 ma 2

IIT-JAM 2011
Q17. The wave function of a quantum mechanical particle is given by

  x   1  x    2  x 
3 4
5 5

where 1  x  and 2  x  are eigenfunctions with corresponding energy eigenvalues

1eV and 2 eV , respectively. The energy of the particle in the state  is


41 11 36 7
(a) eV (b) eV (c) eV (d) eV
25 5 25 5

Ans. : (a)
 H 9 16 41
Solution:  E   = 1eV   2eV  = eV
 25 25 25

H.No. 40-D, Ground Floor, Jia Sarai, Near IIT, Hauz Khas, New Delhi-110016
Phone: 011-26865455/+91-9871145498
Website: www.physicsbyfiziks.com | Email: fiziks.physics@gmail.com

fiziks
Institute for NET/JRF, GATE, IIT‐JAM, M.Sc. Entrance, JEST, TIFR and GRE in Physics 
 
Q18. Light described by the equation 
E   90V / m  sin 6.28 1015 s 1 t  
sin 12.56 1015 s 1  t  is incident on a metal surface. The work function of the metal is

2.0 eV . Maximum kinetic energy of the photoelectrons will be,


(a) 2.14 eV (b) 4.28 eV (c) 6.28 eV (d) 12.56 eV
Ans. : (c)
Solution: K max    W

For given wave maximum kinetic energy is for highest  so   12.56  1015 sec 1

6.6  1034 J s 12.56  1015 s 1 82.8  1019 J


   eV  8.24eV
2 6.28  1.6  1019
K max    W  8.24eV  2eV  6.24eV

IIT-JAM 2012
Q19. Light takes 4 hours to cover the distance from Sun to Neptune. If you travel in a
spaceship at a speed 0.99 c (where c is the speed of light in vacuum), the time (in
minutes) required to cover the same distance measured with a clock on the spaceship will
be approximately
(a) 34 (b) 56 (c) 85 (d) 144
Ans. : (a)

Solution: l  ct0
v2
1  2  c  4  60  60 1 
 0.99c   c  4  60  60  .14 m
2

c c2
c  4  60  60  .14
t min  33.9  34 min
.99c  60
Q20. 60
27 Co is a radioactive nucleus of half-life 2 ln 2  10 8 s . The activity of 10 g of 60
27 Co in
disintegrations per second is,
1 1
(a)  1010 (b) 5  1010 (c)  1014 (d) 5  1014
5 5

H.No. 40-D, Ground Floor, Jia Sarai, Near IIT, Hauz Khas, New Delhi-110016
Phone: 011-26865455/+91-9871145498
Website: www.physicsbyfiziks.com | Email: fiziks.physics@gmail.com

fiziks
Institute for NET/JRF, GATE, IIT‐JAM, M.Sc. Entrance, JEST, TIFR and GRE in Physics 
 
Ans. : (d)

Solution: R  N , where N 
10
60

 6 1023  1023
0.693 0.693 0.693 0.693 0.693
      5  10 9 s 1
T1 / 2 2 ln 2  10 8
2  2.303  0.3010  10 8
1.386  10 8
1.386  10 8

10
 
Thus, R  5 109   6 1023  5 1014 .
60

IIT-JAM 2013
Q21. Electric field component of an electromagnetic radiation varies with time as,
E  acos  0 t  sin t cos  0 t  , where a is a constant and the values of  and  0 are

1  1015 s 1 and 5  1015 s 1 respectively. This radiation falls on a metal of work function
2eV . The maximum kinetic energy (in eV ) of photoelectrons is
(a) 0.64 (b) 1.30 (c) 1.70 (d) 1.95
Ans. : (b)
Solution: K max    W

For given wave, maximum kinetic energy is for highest   , so 0  5  1015 sec 1

6.6  1034 J s 5  1015 s 1 33 1019 J


 0   eV  3.28
2 6.28 1.6  1019
K max    W  3.28eV  2eV  1.28eV

Q22. A free particle of mass m is confined to a region of length L . The de Broglie wave
associated with the particle is sinusoidal in nature as given in the figure. The energy of
the particle is

0
L/3 2L / 3 L
Ans. :
Solution: If wavelength of standing wave is  and length of wall is L then from the figure
3 2L
L   .
2 3

H.No. 40-D, Ground Floor, Jia Sarai, Near IIT, Hauz Khas, New Delhi-110016
Phone: 011-26865455/+91-9871145498
Website: www.physicsbyfiziks.com | Email: fiziks.physics@gmail.com
10 
fiziks
Institute for NET/JRF, GATE, IIT‐JAM, M.Sc. Entrance, JEST, TIFR and GRE in Physics 
 
h
If p is momentum and  is wavelength, then from de-Broglie hypothesis p  , thus

3h
p .
2L
When particle is confined into a box then total energy is only kinetic energy which is
p2 3h 9h 2
given by E  put the value of p  one will get E  .
2m 2L 8mL2

IIT-JAM 2014
Q23. In a photoelectric effect experiment, ultraviolet light of wavelength 320 nm falls on the
photocathode with work function of 2.1 eV. The stopping potential should be close to
(a) 1.8 V (b) 1.6 V (c) 2.2 V (d) 2.4V
Ans. : (a)
hc W
Solution: Since, K .E  eV    W  V  
e e
Now,   320 109 m, W  2.1 eV

6.6 1034  3 108


V   2.1
1.6 1019  320 109
V  3.867  2.1  3.9  2.1  1.8 V .
Thus, option (a) is correct.
Q24. Four particles of mass m each are inside a two dimensional square box of side L . If each
state obtained from the solution of the Schrodinger equation is occupied by only one
h2
particle, the minimum energy of the system in units of is
mL2
5 11 25
(a) 2 (b) (c) (d)
2 2 4
Ans. : (b)
Solution: For 2 - Dimensional box, possible configurations are 1,1 ,  2,1 ,  2, 2 

 22  22
Now, ground state energy  2mL2  x y 
n 2
 n 2
; let E0 
2mL2

H.No. 40-D, Ground Floor, Jia Sarai, Near IIT, Hauz Khas, New Delhi-110016
Phone: 011-26865455/+91-9871145498
Website: www.physicsbyfiziks.com | Email: fiziks.physics@gmail.com
11 
fiziks
Institute for NET/JRF, GATE, IIT‐JAM, M.Sc. Entrance, JEST, TIFR and GRE in Physics 
 
 
2 2
 2 E0 1  2  5E0  1 8E0  20 E0  20 
2mL2
5 2
E
2 mL2
Thus, option (b) is correct
Q25. Two frames, O and O , are in relative motion as shown. c/2
O  is moving with speed c / 2 , where c is the speed of light.
O O
In frame O , two separate events occur at x1 , t1  and x
x2 , t 2 . In frame O , these events occur simultaneously.
The value of  x 2  x1  / t 2  t1  is
(a) c / 4 (b) c / 2 (c) 2c (d) c

x2  vt2
Ans.: (c) x2 
v2
1
c2

x1  vt1 x2  x1


x1   x2  x1 
v2 v2
1 2 1 2
c c

Vn2 vx 
2
t21 t1  21
t2  t2  c ,t 
1
c
2
v v2
1 2 1 2
c c
 v2  c2  v2  c2
x2   t2 1  2  t2   x2  x1   t2  t1  1  2 
   
  v
 c   c  v

v2 c2
 t2  t1  1  2 
x2  x1  c v
2
v
1 2
c
c2 h  h c 2 2c 2
 x2  xy    t2  t1  v
 2 1 
t2  t1 v c
 2c

H.No. 40-D, Ground Floor, Jia Sarai, Near IIT, Hauz Khas, New Delhi-110016
Phone: 011-26865455/+91-9871145498
Website: www.physicsbyfiziks.com | Email: fiziks.physics@gmail.com
12 
fiziks
Institute for NET/JRF, GATE, IIT‐JAM, M.Sc. Entrance, JEST, TIFR and GRE in Physics 
 
IIT-JAM 2015
Q26. A particle with energy E is incident on a potential given by
 0, x0
V  x   .
 V0 , x0
The wave function of the particle for E  V0 in the region x  0 (in terms of positive

constants A, B and k ) is

(a) Ae kx  Be  kx (b) Ae  kx (c) Ae ikx  Be  ikx (d) Zero


Ans. : (b)
 2 d 2 
Solution: For x  0 ;   V0   E  ; E  V0
2m d

2m V0  E 
   Bekx  Ae  kx , where k 
2
   0 as x    A  0     Ae kx
Q27. A system comprises of three electrons. There are three single particle energy levels
accessible to each of these electrons. The number of possible configurations for this
system is
(a) 1 (b) 3 (c) 6 (d) 7
Ans. : (d)
1
Solution: For electron spin is . So in one single state two electrons can be adjusted the number
2
of ways are
Ground First Second
1 2 1 0
2 2 0 1
3 1 2 0
4 1 0 2
5 0 1 2
6 0 2 1
7 1 1 1
So, number of ways are 7.

H.No. 40-D, Ground Floor, Jia Sarai, Near IIT, Hauz Khas, New Delhi-110016
Phone: 011-26865455/+91-9871145498
Website: www.physicsbyfiziks.com | Email: fiziks.physics@gmail.com
13 
fiziks
Institute for NET/JRF, GATE, IIT‐JAM, M.Sc. Entrance, JEST, TIFR and GRE in Physics 
 
Q28. The variation of binding energy per nucleon with respect to the mass number of nuclei is
shown in the figure.
9
8

Average binding energy per 7


nucleon (MeV) 6
5
4
3
2
1
0
20 40 60 80 100 120 140 160 180 200 220 240
Number of nucleons in nucleus, A
Consider the following reactions:
(i) 238
92 U 82
206
Pb  10 P  22n (ii) 238
92 U Pb  8 24He  6e 
206
82

238
Which one of the following statements is true for the given decay modes of 92 U?
(a) Both (i) and (ii) are allowed (b) Both (i) and (ii) are forbidden
(c) (i) is forbidden and (ii) is allowed (d) (i) is allowed and (ii) is forbidden
Ans. : (c)
Solution: In reaction (i) all conservation laws are valid. In reaction (ii) charge is not conserved.
Q29. A nucleus has a size of 10 15 m . Consider an electron bound within a nucleus. The
estimated energy of this electron is of the order of
(a) 1 MeV (b) 10 2 MeV (c) 10 4 MeV (d) 10 6 MeV
Ans. : (d)
h 6.6 1034
Solution: p    6.6  1019 kgm / sec
 1015
p2 44  1038
E    2.4  107 Joule
2me 2  9.1 1031

2.4  107
E 19
eV  1.5  1012 eV  1.5 106 MeV
1.6 10

H.No. 40-D, Ground Floor, Jia Sarai, Near IIT, Hauz Khas, New Delhi-110016
Phone: 011-26865455/+91-9871145498
Website: www.physicsbyfiziks.com | Email: fiziks.physics@gmail.com
14 
fiziks
Institute for NET/JRF, GATE, IIT‐JAM, M.Sc. Entrance, JEST, TIFR and GRE in Physics 
 
Q30. A proton from outer space is moving towards earth with velocity 0.99 c as measured in
earth’s frame. A spaceship, traveling parallel to the proton, measures proton’s velocity to
be 0.97 c . The approximate velocity of the spaceship in the earth’s frame, is
(a) 0.2 c (b) 0.3 c (c) 0.4 c (d) 0.5 c
Ans.: (d)
Solution: Velocity of proton w.r.t. spaceship  0.97 c s s
E p  0.99 c
 u x  0.99 c, v  v, u x  0.97 c v
u x  v 0.99 c  v p  0.99 c
 ux   0.97 c   v  0.5 c
ux v 0.97v
1 2 1
c c
Q31. A particle is moving in a two dimensional potential well
0, 0  x  L, 0  y  2 L
V  x, y   
 , elsewhere

which of the following statements about the ground state energy E1 and ground state
eigenfunction  0 are true?

 2 2 5 2 2
(a) E1  (b) E1 
mL2 8mL2
2 x y 2 x y
(c) 0  sin sin (d)  0  cos cos
L L 2L L L 2L
Ans. : (b) and (c)

 2  2  nx2
n y2 
Solution: En    
2m  L2 4 L2 

 22  1 1  5 2  2
Ground state nx  1, n y  1  Ex    
2m  L2 4 L2  8mL2

2 2 sin  x sin  y
Wave function    
L 2L L 2L

H.No. 40-D, Ground Floor, Jia Sarai, Near IIT, Hauz Khas, New Delhi-110016
Phone: 011-26865455/+91-9871145498
Website: www.physicsbyfiziks.com | Email: fiziks.physics@gmail.com
15 
fiziks
Institute for NET/JRF, GATE, IIT‐JAM, M.Sc. Entrance, JEST, TIFR and GRE in Physics 
 
Q32. Muons are elementary particles produced in the upper atmosphere. They have a life time
of 2.2s . Consider muons which are traveling vertically towards the earth’s surface at a
speed of 0.998c . For an observer on earth, the height of the atmosphere above the
surface of the earth is 10.4 km . Which of the following statements are true?
(a) The muons can never reach earth’s surface
(b) The apparent thickness of earth’s atmosphere in muon’s frame of reference is 0.96 km
(c) The lifetime of muons in earth’s frame of reference is 34.8s
(d) Muons traveling at a speed greater than 0.998 c reach the earth’s surface
Ans.: (c) and (d)
t0 2.2 106
Solution: t   t   34.8 106 sec
1   0.998 
2 2
v
1
c2
Now distance will be  t  v  34.8 106  0.998  3 108  10.4192 km

Apparent thickness X  t  v  2.2 106  0.998  3 108  0.658 km


Q33. A particle is in a state which is a superposition of the ground state  0 and the first

excited state 1 of a one-dimensional quantum harmonic oscillator. The state is given by


1 2
 0  1 . The expectation value of the energy of the particle in this state (in
5 5
units of  ,  being the frequency of the oscillator) is…………
Ans. : 1.3
 1    1  3  4
Solution:  En   n    and P    , P  2   5
 2  2  5
 1 3 4 13
 E       1.3
2 5 2 5 10
Q34. In the hydrogen atom spectrum. the ratio of the longest wavelength in the Lyman series
(final state n  1 ) to that in the Balmer series (final State n  2 ) is…………..
Ans. : 0.185

H.No. 40-D, Ground Floor, Jia Sarai, Near IIT, Hauz Khas, New Delhi-110016
Phone: 011-26865455/+91-9871145498
Website: www.physicsbyfiziks.com | Email: fiziks.physics@gmail.com
16 
fiziks
Institute for NET/JRF, GATE, IIT‐JAM, M.Sc. Entrance, JEST, TIFR and GRE in Physics 
 
1  1 1
Solution: According to Bohr Theory  R 2  2 
L  n f ni  n3

The longest wavelength in the Lyman series is n2


H
1 1 1  3 4
  R   2   R    L 
L 1 2  4 3R n 1
L
The longest wavelength in the Balmer series is
1  1 1 1 1 94 1  5  36
  R 2  2   R    R   R    B 
B 2 3  4 9  36  B  36  5R
L 4 5R 5
     0.185
B 3R 36 27
Q35. X  rays of wavelength 0.24 nm are Compton scattered and the scattered beam is

observed at an angle of 60 o relative to the incident beam. The Compton wavelength of


the electron is 0.00243 nm . The kinetic energy of scattered elections in eV is……………
Ans. : 25
Solution:   0.24 nm, C  0.00243 and   600

      C 1  cos         C 1  cos  

 1 1
    0.24  0.00243 1    0.24  0.00243   0.24  0.00121  0.2412nm
 2 2
Kinetic Energy of scattered electron
hc hc  1 1  1
K .E.    6.6 1034  3 108     9 Joules
   0.24 0.2412  10
19.8  1026 19.8  1026
 K .E.   4.17  4.15    0.02  396 1020 Joules
109 109
396 1020
 K .E.  eV  24.75 eV
1.6 1019

H.No. 40-D, Ground Floor, Jia Sarai, Near IIT, Hauz Khas, New Delhi-110016
Phone: 011-26865455/+91-9871145498
Website: www.physicsbyfiziks.com | Email: fiziks.physics@gmail.com
17 
fiziks
Institute for NET/JRF, GATE, IIT‐JAM, M.Sc. Entrance, JEST, TIFR and GRE in Physics 
 
IIT-JAM 2016
Q36. Consider a free electron  e  and a photon  ph  both having 10 eV of energy. If  and

P represents wavelength and momentum respectively, then


(mass of electron  9.1 1031 kg ; speed of light  3  108 m / s )
(a) e   ph and Pe  Pph (b) e   ph and Pe  Pph

(c) e   ph and Pe  Pph (d) e   ph and Pe  Pph

Ans. : (c)
E h hc
Solution: For photon p ph  ,  ph  
c p E

E 2  m2c 4 h hc
For electron pe  , e  
c p E 2  m2c 4
Q37. A slit has width ‘ d ’ along the x -direction. If a beam of electrons, accelerated in
y -direction to a particular velocity by applying a potential difference of 100  0.1 kV

passes through the slit, then, which of the following statement  s  is (are) correct?

(a) The uncertainty in the position of the electrons in x -direction before passing the slit is
zero

(b) The momentum of electrons in x - direction is  immediately after passing the slit
d
(c) The uncertainty in the position of electrons in y - direction before passing the slit is
zero
(d) The presence of the slit does not affect the uncertainty in momentum of electrons in
y - direction
Ans. : (b) and (d) x
Solution: The electrons beam before slit is Px

collimated in y - direction as shown in Py y
d
figure. Thus, before slit
Py  P and Px  0

H.No. 40-D, Ground Floor, Jia Sarai, Near IIT, Hauz Khas, New Delhi-110016
Phone: 011-26865455/+91-9871145498
Website: www.physicsbyfiziks.com | Email: fiziks.physics@gmail.com
18 
fiziks
Institute for NET/JRF, GATE, IIT‐JAM, M.Sc. Entrance, JEST, TIFR and GRE in Physics 
 
also x   as Px  0
Thus options (a) and (c) are not correct.
 
Now, after the slit Px  d as a result Px  
x d

i.e., Px 
d
Thus, option (b) is correct.
Whereas presence of slit does not affect the uncertainty in momentum in y - direction.
Thus option (d) is also correct.
Q38. A free particle of energy E collides with a one-dimensional square potential barrier of
height V and width W . Which one of the following statement(s) is/are correct?
(a) For E  V , the transmission coefficient for the particle across the barrier will always
be unity
(b) For E  V , the transmission coefficient changes more rapidly with W than with V
(c) For E  V , if V is doubled, the transmission coefficient will also be doubled.
(d) Sum of the reflection and the transmission coefficients is always one
Ans. : (b) and (d)
Solution: R  T  1
2
 E V  E 
R   
 E V  E 
Q39. A particular radioisotope has a half-life of 5 days. In 15 days the probability of decay in
percentage will be…………..
Ans. : 87.5
t / T1/ 2 15/ 5
1 1 N0
N  N0    N0   
2 2 8
N0  N 7
In 15 days the probability of decay  100   100  87.5%
N0 8

H.No. 40-D, Ground Floor, Jia Sarai, Near IIT, Hauz Khas, New Delhi-110016
Phone: 011-26865455/+91-9871145498
Website: www.physicsbyfiziks.com | Email: fiziks.physics@gmail.com
19 
fiziks
Institute for NET/JRF, GATE, IIT‐JAM, M.Sc. Entrance, JEST, TIFR and GRE in Physics 
 
Q40. In photoelectric experiment both sodium (work function  2.3 eV ) and tungsten (work
function  4.5 eV ) metals were illuminated by an ultraviolet light of same wavelength. If
the stopping potential for tungsten is measured to be1.8V , the value of the stopping
potential for sodium will be………….. V .
Ans. : 4
Solution: For tungsten eVs  h  Wt  h  eVs  Wt  1.8  4.5  6.3

For sodium eVs  h  Ws  6.3  2.3  4eV

Q41. The de Broglie wavelength of a relativistic electron having 1 MeV of energy

is………….. 1012 m . (Take the rest mass energy of the electron to be 0.5 MeV . Plank

constant  6.63 1034 Js , speed of light  3 108 m / s , Electronic charge  1.6  1019 C )
Ans. : 1.43

 
2
E 2  m0 c 2 1  .25
  .75MeV
2
Solution: E 2  p 2 c 2  m0 c 2  p 2
 
c c c

h 6.6  1034  3  108 19.8  1013


As,    13
  14.34  1013 m  1.43  1012 m
p .75 1.6  10 1.38
Q42. X -ray of 20 keV energy is scattered inelastically from a carbon target. The kinetic

energy transferred to the recoiling electron by photons scattered at 900 with respect to the
incident beam is……………. keV .
(Planck constant = 6.6 1034 Js, Speed of light = 3 108 m / s, electron mass =

9.11031 kg. Electronic charge = 1.6 1019 C )


Ans. : 0.77
h h 
Solution:       1  cos     '     
mc mc 2
'  1 1 1 1 1 1 1 1 1
   2
 '  2 '  2 
hc hc mc E E mc E E mc 20keV .5MeV

 E '  19.23keV
Recoil velocity of electron E  E '  0.77keV

H.No. 40-D, Ground Floor, Jia Sarai, Near IIT, Hauz Khas, New Delhi-110016
Phone: 011-26865455/+91-9871145498
Website: www.physicsbyfiziks.com | Email: fiziks.physics@gmail.com
20 
fiziks
Institute for NET/JRF, GATE, IIT‐JAM, M.Sc. Entrance, JEST, TIFR and GRE in Physics 
 
IIT-JAM 2017
c
Q43. Consider an inertial frame S  moving at speed away from another inertial frame S
2
along the common x-x axis, where c is the speed of light. As observed from S  , a
c
particle is moving with speed in the y  direction, as shown in the figure. The speed of
2
c/2
the particle as seen from S is:

y y
c/2
S x S x

c c 7c 3c
(a) (b) (c) (d)
2 2 4 5
Ans. : (c)
c c
Solution: v  iˆ u x'  0, u 'y  , u z'  0
2 2

v2 v2
u 'y 1  u '
1 
u'  v c c2  c 1  1  3 c , u 
z
c2  0
ux  x '  , ux  z
uv 2 u' v 2 4 4 u' v
1  x2 1  x2 1  x2
c c c

c 2 3c 2 7c
u  
4 16 4
Q44. Consider Rydberg (hydrogen-like) atoms in a highly excited state with n around 300 .
The wavelength of radiation coming out of these atoms for transitions to the adjacent
states lies in the range:
(a) Gamma rays    pm  (b) UV    nm 

(c) Infrared     m  (d) RF    m 

Ans. : (d)

1  1 1 
Solution:  R 2  2 
  n f ni 
 

H.No. 40-D, Ground Floor, Jia Sarai, Near IIT, Hauz Khas, New Delhi-110016
Phone: 011-26865455/+91-9871145498
Website: www.physicsbyfiziks.com | Email: fiziks.physics@gmail.com
21 
fiziks
Institute for NET/JRF, GATE, IIT‐JAM, M.Sc. Entrance, JEST, TIFR and GRE in Physics 
 
1
where R  1.097 10 m 7

n  300
1  ni2  n 2f  
n f  299 and ni  300   R  2 2  E0
  ni nT 

1  ni n f  1   300   299  
2 2 2 2

   2   
R  ni  n 2f  R   300 2   299 2 
 

1   300 2  299 2  1.34 107


    1.22m    1.22m
1.097  107  599  1.097 107
 
This wavelength corresponds to RF Thus correct option is (d)
Q45. A photon of frequency  strikes an electron of mass m initially at rest. After scattering
at an angle  , the photon loses half of its energy. If the electron recoils at an angle  ,
which of the following is (are) true?
 mc 2 
(a) cos   1  
 h 

 mc 2 
(b) sin    1  
 h 

(c) The ratio of the magnitudes of momenta of the recoiled electron and scattered photon
sin 
is
sin 
h
(d) Change in photon wavelength is 1  2 cos  
mc
Ans. : (a), (c)
h c c h
1  cos       1  cos  
Solution:     
mc   mc
2c c h c h  mc 2 
   1  cos     1  cos    cos   1  
  mc  mc  h 

From the conservation of momentum in y direction


h  p sin 
sin   p sin   
c h  sin 
c

H.No. 40-D, Ground Floor, Jia Sarai, Near IIT, Hauz Khas, New Delhi-110016
Phone: 011-26865455/+91-9871145498
Website: www.physicsbyfiziks.com | Email: fiziks.physics@gmail.com
22 
fiziks
Institute for NET/JRF, GATE, IIT‐JAM, M.Sc. Entrance, JEST, TIFR and GRE in Physics 
 
Q46. For an atomic nucleus with atomic number Z and mass number A , which of the
following is (are) correct?
(a) Nuclear matter and nuclear charge are distributed identically in the nuclear volume
(b) Nuclei with Z  83 and A  209 emit  - radiation
(c) The surface contribution to the binding energy is proportional to A2 / 3
(d)  - decay occurs when the proton to neutron ratio is large, but not when it is small
Ans. : (b) and (c)
Solution: From given statement only (b) and (c) are correct.
Q47. Consider a one-dimensional harmonic oscillator of angular frequency  . If 5 identical
particles occupy the energy levels of this oscillator at zero temperature, which of the
following statement(s) about their ground state energy E0 is (are) correct?

13
(a) If the particles are electrons, E0  
2
25
(b) If the particles are protons, E0  
2
25
(c) If the particles are spin-less fermions, E0  
2
5
(d) If the particles are bosons, E0  
2
Ans. : (a), (c) and (d)
Solution: If particles are electrons and protons then ground state energy
 3 5 13
E0  2   2  1 
2 2 2 2
If the particles are spin-less fermions, then energy is
 3 5 7 9 25
E0      
2 2 2 2 2 2
1 5
If the particles are bosons E0  5   
2 2

H.No. 40-D, Ground Floor, Jia Sarai, Near IIT, Hauz Khas, New Delhi-110016
Phone: 011-26865455/+91-9871145498
Website: www.physicsbyfiziks.com | Email: fiziks.physics@gmail.com
23 
fiziks
Institute for NET/JRF, GATE, IIT‐JAM, M.Sc. Entrance, JEST, TIFR and GRE in Physics 
 
Q48. A particle of mass m is placed in a three-dimensional cubic box of side a . What is the
  2 2 
degeneracy of its energy level with energy 14  2 
?
 2ma 
(Express your answer as an integer)
Ans. : 6
Solution: nx2  n 2y  nz2  14

nx  1, n y  2, nz  3
nx  1, n y  3, nz  2
nx  2, n y  3, nz  1
nx  2, n y  3, nz  1
nx  3, n y  1, nz  2
nx  3, n y  2, nz  1

So degeneracy is 6
Q49. For a proton to capture an electron to form a neutron and a neutrino (assumed massless),
the electron must have some minimum energy. For such an electron the de-Broglie
wavelength in pictometers is……………
(Specify your answer to two digits after the decimal point)
Ans. : 1.02
Solution: From conservation of energy

Ee  me c 2  K e   mn  m p  c 2  1.675  1.673 1027  3  108   1.8  1013 Joules


2

Ee
Ee2   pc   me2 c 4   pc   p  0.6 1022 kg.m / sec  pc  me c 2 
2 2

c
h 6.6  1034
  22
 1.1 1012 m  1.1 pm
p 0.6  10

H.No. 40-D, Ground Floor, Jia Sarai, Near IIT, Hauz Khas, New Delhi-110016
Phone: 011-26865455/+91-9871145498
Website: www.physicsbyfiziks.com | Email: fiziks.physics@gmail.com
24 
fiziks
Institute for NET/JRF, GATE, IIT‐JAM, M.Sc. Entrance, JEST, TIFR and GRE in Physics 
 
IIT-JAM 2018
Q50. Let Tg and Te be the kinetic energies of the electron in the ground and the third excited

Tg
states of a hydrogen atom, respectively. According to the Bohr model, the ratio is
Te
(a) 3 (b) 4 (c) 9 (d) 16
Ans. : (d)
Solution: From Bohr model the kinetic energy and Total energy E and kinetic energy T

E E E T E 16
T  where Eg  0 , Ee  0  g  g   16 :1
2 1 16 Te Fe 1

Q51. The mean momentum p of a nucleon in a nucleus of mass number A and atomic
number Z depends on A, Z as
1 1 1
     2
(d) p   AZ  3

(a) p  A 3 (b) p  Z 3 (c) p  A 3
Ans. : (c)

Solution: The radius of a nucleus can be combined as (greater than the wavelength of
2
electron)
h
The moment p 

h
  R  R0 A1/ 3 which implies p   A1/ 3 .
R0
As, p  A1/ 3
 0, 0  x  L
Q52. A particle of mass m is in a one dimensional potential V  x    .
, otherwise
1 2
At some instant its wave function is given by   x    1  x   i  2  x  , where
3 3

 1  x  and  2  x  are the ground and the first excited states, respectively. Identify the
correct statement.
L  2 3 2 2L 2  2
(a) x  ; E  (b) x  ; E 
2 2m L2 3 2m L2
L  2 8 2 2L  2 4 2
(c) x  ; E  (d) x  ; E 
2 2m L2 3 2m 3L2

H.No. 40-D, Ground Floor, Jia Sarai, Near IIT, Hauz Khas, New Delhi-110016
Phone: 011-26865455/+91-9871145498
Website: www.physicsbyfiziks.com | Email: fiziks.physics@gmail.com
25 
fiziks
Institute for NET/JRF, GATE, IIT‐JAM, M.Sc. Entrance, JEST, TIFR and GRE in Physics 
 
Ans.: (a)
1 2 9 E0
  E0   4 E0
 22
Solution: E  3 3  3  3E0 Where, E0 
1 2

3 2mL2
3 3 3
3   2  2 3 2  2
E  
2mL2 2mL2
1 2 i 2 i 2
X  1 X 1   2 X 1  1 X  2   2 X 1
3 3 3 3 3 3
1L 2L L
  
32 32 2
Q53. A system of 8 non-interacting electrons is confined by a three dimensional potential
1
V r   m 2 r 2 . The ground state energy of the system in units of  is ______
2
(Specify your answer as an integer.)
Ans. : 18
Solution: n  0 is non degenerate so there will 2 electron in the ground state.
n  1 is triple degenerate so there is 6 electron in the first excited state
3 5
E  2  6  3  15  18
2 2
Q54. Rod R1 has a rest length 1m and rod R2 has a rest length of 2 m . R1 and R2 are moving

with respect to the laboratory frame with velocities  viˆ and viˆ , respectively. If R2 has

v
a length of 1m in the rest frame of R1 , is given by__________
c
(Specify your answer upto two digits after the decimal point)
Ans. : 0.48
Solution: S S
v
v
R1 v
R2

H.No. 40-D, Ground Floor, Jia Sarai, Near IIT, Hauz Khas, New Delhi-110016
Phone: 011-26865455/+91-9871145498
Website: www.physicsbyfiziks.com | Email: fiziks.physics@gmail.com
26 
fiziks
Institute for NET/JRF, GATE, IIT‐JAM, M.Sc. Entrance, JEST, TIFR and GRE in Physics 
 
V  v , ux  v

ux  V 2v
ux  

uV v2
1  x2 1 2
c c

ux 2
l  l0 1 
c2

ux 2
1  2 1
c2
 4v 2 
 
1  1  v2 / c2  4v 2 / c 2 3  v2  3 3  v2 
 1    4  2   2
4 c2 v2 4 c  4 4c 
1 2
c
2 2
13  v  3  v  12 v 12 v
          0.479  0.48 .
4 c 4  c  52 c 52 c
Q55. Two events E1 and E2 take place in an inertial frame S with respective time space

 
coordinates (in SI units): E1  t1  0, r1  0  and E2 t2  0, x2  108 , y z  0, z2  0 . Another

inertial frame S  is moving with respect to S with a velocity v  0.8 ciˆ . The time

difference  t2  t1  as observed in S  is ___________ s .

(Specify your answer in seconds upto two digits after the decimal point)
Ans. : 0.44
Solution: t2  t1  0 and x2  x1  108

 t2  t1  x x v
t2  t1   2 1


1  v / c2  1  v / c2
2 2  c2

v
 x2  x1 
2
c 
v
x 2  x1 2
c 
 
0.8c
108  2
c  8  10
8
8
   0.44sec .
1  0.64 6  3  10
8
1 v / c
2 2
1 v / c
2 2 18

H.No. 40-D, Ground Floor, Jia Sarai, Near IIT, Hauz Khas, New Delhi-110016
Phone: 011-26865455/+91-9871145498
Website: www.physicsbyfiziks.com | Email: fiziks.physics@gmail.com
27 
fiziks
Institute for NET/JRF, GATE, IIT‐JAM, M.Sc. Entrance, JEST, TIFR and GRE in Physics 
 
IIT-JAM 2019
Q56. A classical particle has total energy E . The plot of
U
potential energy U  as a function of distance  r 
I III
from the centre of force located at r  0 is shown in E IV
II
the figure. Which of the regions are forbidden for
the particle? 0 r

(a) I and II (b) II and IV


(c) I an IV (d) I and III
Ans. : (d)
Solution: In the region I and III potential energy is more than total energy .
235
Q57. In the thermal neutron induced fission of U , the distribution of relative number of the
observed fission fragments (Yield) versus mass number  A  is given by

(a) (b) (c) (d)


Yield

Yield

Yield

Yield
80 120 160 80 120 160 80 120 160 80 120 160
A A A A
Ans. : (a)
Q58. For a quantum particle confined inside a cubic box of side L , the ground state energy is
given by E0 . The energy of the first excited state is

(a) 2E0 (b) 2E0 (c) 3E0 (d) 6E0


Ans. : (d)

n 2

 n 2y  nz2  2  2
Solution: Enx ,n y ,nz 
x

2ma 2 
 nx2  n 2y  nz2 E0 
E2,1,1  E1,2,1  E1,1,2 
 4  1  1  2 2  6E 0
2ma 2

H.No. 40-D, Ground Floor, Jia Sarai, Near IIT, Hauz Khas, New Delhi-110016
Phone: 011-26865455/+91-9871145498
Website: www.physicsbyfiziks.com | Email: fiziks.physics@gmail.com
28 
fiziks
Institute for NET/JRF, GATE, IIT‐JAM, M.Sc. Entrance, JEST, TIFR and GRE in Physics 
 
Q59. A  -ray photon emitted from a Cs source collides with an electron at rest. If the
137

Compton shift of the photon is 3.25 1013 m , then the scattering angle is closets to

(Planck’s constant h  6.626  1034 Js , electron mass m  9.109 1031 kg and velocity

of light in free space c  3 108 m/s )

(a) 45o (b) 60o (c) 30o (d) 90o


Ans. : (c)
h  .me c
Solution:   1  cos    cos   1 
me c h

3.25  1013  9.109  1031  3  108 3


 1 34
 0.866 
6.6  10 2

  300
Q60. The relation between the nuclear radius  R and the mass number  A  , given

by R  1.2 A1/ 3 fm , implies that


(a) The central density of nuclei is independent of A
(b) The volume energy per nucleon is a constant
(c) The attractive part of the nuclear force has a long range
(d) The nuclear force is charge dependent
Ans. : (a), (b), (d)
Q61. An atomic nucleus X with half-life TX decays to a nucleus Y , which has half-life TY .
The condition (s) for secular equilibrium is (are)
(a) TX  TY (b) TX  TY (c) TX  TY (d) Tx  TY
Ans. : (d)
Q62. In a typical human body, the amount of radioactive 40
K is 3.24 105 percent of its mass.
40
The activity due to K in a human body of mass 70 kg is ______ kBq.
(Round off to 2 decimal places)
(Half-life of 40
K  3.942 1016 S , Avogadro’s number N A  6.022 1023 mol1

Ans. : 6.0

H.No. 40-D, Ground Floor, Jia Sarai, Near IIT, Hauz Khas, New Delhi-110016
Phone: 011-26865455/+91-9871145498
Website: www.physicsbyfiziks.com | Email: fiziks.physics@gmail.com
29 
fiziks
Institute for NET/JRF, GATE, IIT‐JAM, M.Sc. Entrance, JEST, TIFR and GRE in Physics 
 
dN
Solution:  N
dt


0.693

 

70 103 3.24 105
 6.022  10
3.942 106  s  40 100

 6.0  1013 disintegrations / s


 6.0  1013 Bq
 6.0  1010 kBq
Q63. A proton is confined within a nucleus of size 1013 cm . The uncertainty in its velocity is

_________ 108 m/s .


(Round off to 2 decimal places)
(Planck’s constant h  6.626 1034 J and proton mass mP  1.672 1027 kg )

Ans. : 0.31
h
Solution:  p  x 
4
h 6.6  1034
v    0.31108 m / s
4 m  x 4  3.14  1.672  10  10
27 15
 
2  
Q64. Given the wave function of a particle   x   sin  x  0  x  L and 0 elsewhere
L L 
L
the probability of finding the particle between x  0 and x  is _______.
2
(Round off to 1 decimal places)
Ans. : 0.5

2    L  L/2 2 1
Solution:   x   sin  x  0  x  L , p  0  x      dx 
L L   2 0 2

H.No. 40-D, Ground Floor, Jia Sarai, Near IIT, Hauz Khas, New Delhi-110016
Phone: 011-26865455/+91-9871145498
Website: www.physicsbyfiziks.com | Email: fiziks.physics@gmail.com
30 

You might also like